Given the function f(x) = 3x + 1 and the linear function g(x), which function has a greater value when x = 3?

graph of line g of x equals 4 times x minus 5

f(x) is greater.
g(x) is greater.
f(x) and g(x) are the same when x = 3.
g(x) is undefined when x = 3.

Answers

Answer 1

Answer:

f(x) is greater

Step-by-step explanation:

Answer 2

The function that has a greater value when x = 3 is: A. f(x) is greater.

What is a linear function?

In Mathematics and Geometry, a linear function refers to a type of function whose equation is graphically represented by a straight line on the xy-plane or cartesian coordinate.

Based on the information provided, we can logically deduce the following linear functions;

f(x) = 3x + 1

g(x) = 4x - 5

When x is equal to 3, we have:

f(x) = 3x + 1

f(3) = 3(3) + 1

f(3) = 10

g(x) = 4x - 5

g(3) = 4(3) - 5

g(3) = 7.

Therefore, f(x) is greater.

Read more on linear function here: brainly.com/question/27325295

#SPJ3


Related Questions

Please answer this correctly

Answers

Gary swam the fewest laps.
Hope this helps!

Answer:

Gary

Step-by-step explanation:

Edward more than Gabby

So Edward>Gabby

But Edward swam fewer laps than Noelle

So Noelle>Edward>Gabby

Gabby swam more than Gary

So Noelle>Edward>Gabby>Gary

what do the word enlightened mean​

Answers

Answer:

To understand something or someone completely

Step-by-step explanation:

It originates from the word enligtment

Solve for x: 6x + 3 = 5x − 8

Answers

Answer:

x = -11

Step-by-step explanation:

First subtract 5x from both sides (6x - 5x) and the equation turns into: x + 3 = -8. Then subtract 3 from both sides, -8 - 3, and that equals -11. And so the final answer is -11.

The value of x from the equation 6x + 3 = 5x − 8 is -11.

What is a system of equations?

A system of equations is two or more equations that can be solved to get a unique solution. the power of the equation must be in one degree.

Given equation ; 6x + 3 = 5x − 8

Solve for x:

6x + 3 = 5x − 8

First subtract 5x from both sides

6x + 3 - 5x = − 8

x + 3 = -8

Then subtract 3 from both sides;

x = -8 - 3

x = -11

Thus, the final answer is -11.

Therefore, The value of x from the equation 6x + 3 = 5x − 8 is -11.

Learn more about equations here;

https://brainly.com/question/10413253

#SPJ2

This year, a small business had a total revenue of $45,000. If this is 25% less than their total revenue the previous year, what was their total revenue the previous year?

Answers

Let R = total revenue the year before

R = 45,000 x 0.25 + 45,000

R = 11,250 + 45,000

R = $56,250

Steps to solve the equation
-2(x-4)+8=2

Answers

Answer:

x=-1

Step-by-step explanation:

-2(x+4)+8=2

1) Use the Distributive property:

-2x-8+8=2

2) Combine alike terms:

-2x=2

3) Divide both sides by -2:

x=-1

10. A movie theater advertises that a family of two adults, one student, and one child between the ages of 3
and 8 can attend a movie for $15. An adult ticket costs as much as the combined cost of a student ticket
and a child ticket. You purchase 1 adult ticket, 4 student tickets, and 2 child tickets for $23.

Answers

Answer:

Students ticket = $4

Child ticket = $1

Adult ticket = $5

Step-by-step explanation:

Let the price of the student ticket be $x and the price of a child ticket be $y.  An adult ticket costs as much as the combined cost of a student ticket and a child ticket, so the price of one adult ticket is $(x+y).

1. A movie theater advertises that a family of two adults, one student, and one child between the ages of 3 and 8 can attend a movie for $15. Then

[tex]2(x+y)+x+y=15[/tex]

2. If you purchase 1 adult ticket, 4 student tickets, and 2 child tickets for $23, then

[tex](x+y)+4x+2y=23[/tex]

Now, solve the system of two equations:

[tex]\left\{\begin{array}{l}2(x+y)+x+y=15\\ \\(x+y)+4x+2y=23\end{array}\right.\Rightarrow \left\{\begin{array}{l}3(x+y)=15\\ \\5x+3y=23\end{array}\right.\\ \\\left\{\begin{array}{l}x+y=5\\ \\5x+3y=23\end{array}\right.\\ \\\left\{\begin{array}{l}y=5-x\\ \\5x+3(5-x)=23\end{array}\right.[/tex]

Solve the last equation

[tex]5x+15-3x=23\\ \\2x=23-15\\ \\2x=8\\ \\x=4\\ \\y=5-x=5-4=1[/tex]

Students ticket = $4

Child ticket = $1

Adult ticket = $5

Answer:

The ticket prices will as follows:

Students ticket = $4

Child ticket = $1

Adult ticket = $5

Step-by-step explanation:

Thinking process:

Let the price of the student ticket be $x and the price of a child ticket be $y.  An adult ticket costs as much as the combined cost of a student ticket and a child ticket, so the price of one adult ticket is $(x+y).

1. A movie theater advertises that a family of two adults, one student, and one child between the ages of 3 and 8 can attend a movie for $15. Then

2. If you purchase 1 adult ticket, 4 student tickets, and 2 child tickets for $23, then

Now, solve the system of two equations:

Solve the last equation

Students ticket = $4

Child ticket = $1

Adult ticket = $5

The following table lists the masses, in grams, of the eight planets and the dwarf planet Pluto. Use the data to choose the correct answers in the item below.

Uranus has a mass that is approximately___ times greater than Mercury's.
1. 3,000
2. 300
3. 30
4. 3

Uranus's mass = 8.61 x 10^28
Mercury's mass = 3.302 x 10^26


Answers

Answer:

Option 2) 300

Uranus has a mass that is approximately 300 times greater than Mercury's

Step-by-step explanation:

Let

x ----> Uranus's mass

y ---->  Mercury's mass

we have

[tex]x=8.61*10^28\ units[/tex]

[tex]y=3.302*10^26\ units[/tex]

Divide Uranus's mass by Mercury's mass

[tex]\frac{x}{y}[/tex]

substitute the values

[tex]\frac{8.61*10^28}{3.302*10^26}=260.75[/tex]

Round to the nearest hundred

therefore

Uranus has a mass that is approximately 300 times greater than Mercury's

Answer:

The answer is 300

Step-by-step explanation:

37 +(92÷3) what does it equal??​

Answers

The answer 67.6 and in fraction form it is 203/3

How do you write 549,867 in expanded form?

Answers

In expanded form, 549,867 would be written like this: 500,000 + 40,000 + 9,000 + 800 + 60 + 7. This is how it is supposed to be.

Hope this helped!

Nate

Answer:

(5x100,000) + (4 x 10,000) + (9 x 1,000) + (8 x 100) + (6 x 10) + (7 x 1)

Step-by-step explanation:

multiply the number by the place value.

What is the solution of 12b=18

Answers

Answer:

b=18/12

b=3/2

so practice more

Pencils cost $0.24 each , and pens cost $.79 each. Mrs. Trevino bought 6 pencils and 5 pens. How much did she pay for the pencils and pens in dollars and cents?

Answers

Answer:

$5.39

Step-by-step explanation:

5 x .79 = 3.95

6 x .24 = 1.44

3.95 + 1.44 = 5.39

Answer:

For the pencils, she paid: $1.44

For the pens, she paid: $3.95

Step-by-step explanation:

Pencils cost $0.24each.

Pens cost $0.79 each.

Mrs. Trevino bought 6 pencils and 5 pens.

For the pencils, she paid: 6 × $0.24 = $1.44

For the pens, she paid: 5 × $0.79 = $3.95

Phillip write an integer that is less than -2 and greater than -3.5 Which integer did he write?​

Answers

The answer would be -4

Answer:

The integer he wrote could be between -3.4 to -2.1, you could just say -3

PLEASE HELP AND EXPLAIN!!!
I WILL MARK BRAINLIEST AND THANK YOU!!!

If DF = 42, find DE.

Answers

Answer:  The required length of DE is 29.

Step-by-step explanation:  We are given that DF = 42 in the figure shown.

We are to find the length of DE.

From the figure, we note that

[tex]DE=7x+1,~~EF=4x-3.[/tex]

According to the given information, we have

[tex]DF=42\\\\\Rightarrow DE+EF=42\\\\\Rightarrow 7x+1+4x-3=42\\\\\Rightarrow 11x=42+2\\\\\Rightarrow 11x=44\\\\\Rightarrow x=\dfrac{44}{11}\\\\\Rightarrow x=4.[/tex]

Therefore, we get

[tex]DE=7x+1=7\times4+1=28+1=29.[/tex]

Thus, the required length of DE is 29.

The length of DE using the measurements DF = 42, DE = 7x + 1 and EF = 4x - 3 is 29 unit.

given that,

the length of DF = 42

length of DE = 7x + 1

and, length of EF = 4x - 3

Now, from the figure

DE + EF = DF

Plugging the values back to the formula as

7x + 1 + 4x - 3= 42

Combine like terms

11x - 2 = 42

add 2 both side

11x = 42 + 2

11x = 44

Divide both side by 11

x= 44/11

x= 4

So, the length of DE is

= 7x + 1

= 28 + 1

= 29

Learn more about Equation here:

https://brainly.com/question/29657992

#SPJ3

A pet shop has 562 gold fish. They use tanks that hold up to 18 fish in each. How many tanks will he need to buy to fit all the goldfish?​

Answers

Answer:

32 tanks

Step-by-step explanation:

since 562/18

is 31.222222...

You would have to round up because you the limit is 18 fish. You would need 32 tanks to fit all the fish

Answer:

32

Step-by-step explanation:

You have 562 gold fish and you need to have 18 in each tank, how many tanks do you need?

You first need to start with dividing the gold fish amount to the amount that can fit in each tank, (562 divided by 18 = 31.22222222222222). Then you multiply 18 by 31 and you get 558. This is obviously not as much goldfish as you had before, so to fix this you add another tank which makes 32 tanks, but you minus 562 from 558 and you have only 4 fish in the last tank.

Hope this helps! :)

I am a 2-digit number. The sum of my digits is 11. I am divisible by both 4 and 7

Answers

Answer:

56 or 84

Step-by-step explanation:

Answer:

It's 56

Step-by-step explanation:

We have 2 digit number the sum of digits is 11  and it is divisible by 4 and 7 so, first possibility is 4×7=28 ; but the sum of digits is 11; so,the sum of                digits is 2+8=10 which does not satisfies the first constraint next possible 2 digit number is 56: since the sum of digits is 5+6=11, 56 ÷ 7 = 8 , 56 ÷ 4 = 9

Estefani‘s house is at point E (3,-2) and Jasmin’s house is at point J (-5,3). Jasmin’s house is the Mid-point of Estefani’s house and Preston’s house. Give the y-coordinate of Preston’s house.

Answers

The formula for midpoint is

([tex]\frac{x_{1}+x_{2}}{2}[/tex], [tex]\frac{y_{1}+y_{2}}{2}[/tex])

Look at the image below to see the line segment of Estefani's (A), Jasmin's (M), and Preston's (B) houses. Keep in mind that the segment shown below is not accurate in regards to how the line segment formed by Estefani's, Jasmin's, and Preston's houses. It is simply there so you can picture the segment better.

In this case:

[tex]x_{1} =3\\x_{2} =unknown\\y_{1} =-2\\y_{2} =unknown[/tex]

^^^Plug in these number into the formula given above...

([tex]\frac{3+x}{2}[/tex], [tex]\frac{-2 + y}{2}[/tex])

To find what x is (a coordinate of Preston's house) you must take the x-value part of the midpoint equation ([tex]\frac{3+x}{2}[/tex]) and set it equal to the x-value of the midpoint (-5). Then you must solve for x:

[tex]\frac{3+x}{2}[/tex] = -5

3 + x = -5 * 2

3 + x = -10

x = -10 - 3

x = -13

To find what y is (a coordinate of Preston's house) you must take the  y-value part of the midpoint equation([tex]\frac{-2 + y}{2}[/tex]) and set it equal to the y-value of the midpoint (3). Then you must solve for y:

[tex]\frac{-2 + y}{2}[/tex] = 3

-2 + y = 3 * 2

-2 + y = 6

y = 6 + 2

y = 8

The coordinate of Preston's house is:

(-13, 8)

Hope this helped!

~Just a girl in love with Shawn Mendes

Find the domain of the function.
g(x) = 1 - 2x^2

Answers

The domain of the function will be Domain: ( − ∞ , ∞ ) , { x | x  ∈  R }.

What is the domain of the function?

The entire range of independent variable values is the domain of a function. In algebra, a polynomial function with one or more variables in which the highest-degree term is of the second degree is known as a quadratic function, a quadratic polynomial, a polynomial of degree 2, or simply a quadratic.

The given quadratic function g(x) = 1 - 2x² will have a domain from negative infinity to positive infinity. The graph of the function is also attached with the answer below.

To know more about the domain follow

https://brainly.com/question/26098895

#SPJ2

Which statement is true?


A. Figure A is congruent to figure B.
B. Figure B is congruent to figure C.
C. Figure C is congruent to figure D.
D. Figure D is congruent to figure A.

Answers

Answer:

B. Figure B is congruent to figure C.

Step-by-step explanation:

Figure D is dilated about a quarter of the other two, and Figure A is out of the question because it is dilated about twice their size, so with that being said, you have your answer.

I am joyous to assist you anytime.

Answer:

b

Step-by-step explanation:

Convert the following Celsius degrees to Fahrenheit 35° F

Answers

Answer:

Step-by-step explanation:

95 degrees farighite

2.4 = m + 3.7 linear equations solving

Answers

Answer:

m = -1.3

Step-by-step explanation:

subtract 3.7 from both side so it gets you 2.4-3.7 = m

then you will get -1.3

The answer should be M=-1.3

How do you solve this? Please help

Answers

So (m o n)(8)= m(n(8)). n(8)=-1 and m(-1)=0. So (m o n)(8)=0

x/5 -8=4 find the value of x

Answers

Answer:

x=60

Step-by-step explanation:

I'm assuming you meant this: (x/5)-8=4

In which case you would add 8 to both sides to get rid of the 8 on the left (your goal is to get x by itself so you want to move the numbers on the x side to the other side of the equal sign)

(x/5)=12

Then you would multiply 5 on both sides to get rid of the fraction with the 5 on the bottom on the left side.

x=60

There's your answer.

The formula for the volume V of a cylinder is V = πr2h, where r is the radius of the base and h is the height of the cylinder. Solve the formula for h.


Choose the correct formula.


A. h=Vπr2

B. h=Vπr2

C. h=Vr2π

D. h=Vπr2


Part B

Find the height of a cylinder with a volume of 36π cm3 and a base with a radius of 3 cm.


h = __ cm

Answers

Answer:

Part A) [tex]h=V/(\pi r^{2})[/tex]

Part B) [tex]h=4\ cm[/tex]

Step-by-step explanation:

Part A)

we have the formula of the volume of a cylinder

[tex]V=\pi r^{2}h[/tex]

Solve for h

That means ----> isolate the variable h

Divide both side by πr²

[tex]V/(\pi r^{2})=\pi r^{2}h/(\pi r^{2})[/tex]

Simplify

[tex]V/(\pi r^{2})=h[/tex]

Rewrite

[tex]h=V/(\pi r^{2})[/tex]

Part B) Find the height of a cylinder with a volume of 36π cm3 and a base with a radius of 3 cm

we have

[tex]V=36\pi\ cm^3[/tex]

[tex]r=3\ cm[/tex]

substitute in the formula and solve for h

[tex]h=36\pi/(\pi 3^{2})[/tex]

[tex]h=4\ cm[/tex]

Final answer:

To solve for h in the formula V = πr2h, divide both sides of the equation by πr2 to isolate h. The height of a cylinder with a volume of 36π cm3 and a base radius of 3 cm is 4 cm.

Explanation:

To solve the formula V = πr2h for h, we need to isolate h on one side of the equation. To do this, divide both sides of the equation by πr2. This gives us the equation h = V/(πr2). Therefore, the correct formula for solving for h is h = V/(πr2).

To find the height of a cylinder with a volume of 36π cm3 and a base radius of 3 cm, substitute the given values into the formula h = V/(πr2). This gives us h = 36π / (π * 32). Simplifying further, h = 36 / 9 = 4 cm.

Learn more about Solving for the height of a cylinder here:

https://brainly.com/question/35914373

#SPJ3

Which two whole numbers lies between the square root of 210?

Answers

Answer: 6√35

Step-by-step explanation:

since the √ 210 cannot come out into an equal number then you have to try to get two numbers closest to answering 210 to get it except you have to do 6 square root of 35. I hope this helps you!!!

Final answer:

The two whole numbers between which the square root of 210 lie are 14 and 15.

Explanation:

The square root of 210 is approximately 14.49. Since you are looking for the two whole numbers between which this value lies, you need to find the next higher and lower whole numbers from this value. These would be 14 and 15, as 14.49 is more than 14 and less than 15.

Learn more about Square Root here:

https://brainly.com/question/1540542

#SPJ2

What is -14=3x+x-2 can someone please help

Answers

Answer:

[tex]\large\boxed{x=-3}[/tex]

Step-by-step explanation:

[tex]-14=3x+x-2\qquad\text{add 2 to both sides}\\\\-14+2=3x+x-2+2\qquad\text{combine like terms}\\\\-12=4x\qquad\text{divide both sides by 4}\\\\\dfrac{-12}{4}=\dfrac{4x}{4}\\\\-3=x\to x=-3[/tex]

2 1/8 x 2 6/9

(Multiplying fractions)

Answers

Answer:

17/6

Step-by-step explanation:

Start off with

2 1/8 * 2 6/9

Convert mixed number into improper fraction

17/8

reduce fraction with 3

17/8 * 2 * 2/3

Reduce numbers with GCD 2

17/4 * 2/3

Repeat

17/2 * 1/3

Multiply fractions

17*1/2*3

You then get...

17/6!

[Alternative forms: 2 5/6, 2.83]

A blueprint of a shopping complex shows the bottom edge of the roof to be 93 feet above the ground. If the roof
rises to a point 171 feet above the ground over a horizontal distance of 6.5 yards, what is the slope of the roof?
12
O
1.2
08

Answers

Slope = (rise) / (run)

Rise = (171-93) = 78 ft

Run = 6.5 yards = 19.5 ft

Slope = (78 / 19.5)

Slope = 4

The correct figure is not included in the list of choices.

The slope is 4.

What is slope?

The slope is defined as the ratio of the vertical change between two points, the rise, to the horizontal change between the same two points, the run.

given:

roof rises= 171 feet above the ground

horizontal distance = 6.5 yards

using,

Slope = (rise) / (run)

Rise

= (171-93)

= 78 ft

Run

= 6.5 yards

= 19.5 ft

Slope

= (78 / 19.5)

= 4

hence, the slope is 4.

Learn more about slope here:

https://brainly.com/question/14548961

#SPJ2

2y-3=3y

i need help

Answers

Answer:

y = -3

Step-by-step explanation:

First, subtract 2y from both sides, (3y - 2y) and that should give you the equation, -3 = y. And that's your final answer.

2y-3=3y

Move 2y to the other side

Sign changes from +2y to -2y

2y-2y-3=3y-2y

-3=y

Answer: y=-3

720/91.2 = c/0.513 pleasse solve

Answers

720

91.2

=

c

0.513

Answer:

c

=

4.05

Answer:

c =  4.05  

Step-by-step explanation:

Given: 720/91.2 = c/0.513 .

To find: solve

Solution: We have given that  [tex]\frac{720}{91.2}[/tex]= [tex]\frac{c}{0.513}[/tex]    

 On cross multiplication

 [tex]\frac{720}{91.2}[/tex]= [tex]\frac{c}{0.513}[/tex]

91.2 * c = 0.513 * 720  

91.2 * c = 369.36

On dividing both side by 91.2

 c = [tex]\frac{369.36}{91.2}[/tex]

c =  4.05  

Therefore ,  c =  4.05  .

If a rectangle has a side with vertices located at (0, -8) and (10,-8), the length of the side is:

Answers

Answer:

10 units.

Step-by-step explanation:

That would be the difference in the x coordinates as the y coordinates are both -8.

So it's 10 - 0 = 10 units.

Other Questions
Assume a 8x1 multiplexers data inputs have the following present values: i0=0, i1=0, i2=0, i3=0, i4=0, i5=1, i6=0, i7=0. What should be value of the select inputs s2, s1 and s0 for the value on the multiplexers output d to be 1?s2=s1=s0 8. If a car is going 10 m/s and 4 seconds later it is going 18 m/s, what is its acceleration! Burglar Bob breaks into Vince Victims house. Bob steals a flat-screen television and laptop and does a significant amount of damage to the property before he leaves. Fortunately, Vince has a state-of-the-art security system. It captures excellent images of Bob, who is soon caught by police. Assume that two legal actions follow, one civil and one criminal. Who will be responsible for bringing the civil case? What will be the outcome if the jury believes that Bob burgled Vinces house? Who will be responsible for bringing the criminal case? What will be the outcome if the jury believes that Bob burgled Vinces house? A room with dimensions of 3 x 10 x 20 m is estimated to have outdoor air brought in at an infiltration rate of 1/4 volume changes per hour. Determine the infiltration rate in m^3/s. Is V72 rational or irrational? Explain. The solubility of acetanilide is 0.53 g in 100 mL of icecold water, and 5.50 g in 100 mL of boiling water. What is the maximum percent recovery that can be achieved for the recrystallization of acetanilide from water? 10) At 9:00 there are exactly 10 bacteria in a dish, and the population doubles every 15 minutes . Pick a time after 10:30 but before 12:00 and determine the number of bacteria in the dish at that time. Show all work supporting your answer. What is one advantage of using an image of an ordinary-looking person sitting in a jail cell on an anti-drunk driving poster? The image shows that only people with prison records drive under the influence of alcohol. The image indicates that only the people in prison are those who drive drunk. The image strongly suggests that those who drive drunk may end up serving time in prison. The image powerfully demonstrates that even people in prison know not to drive drunk. Kelton Inc. produces large quantities of products for their customers within tight deadlines. Recently, customers have been returning their products to Kelton, citing errors and defects in the items they have received. Which of the following total quality management cores values does Kelton need to employ to correct this issue? what is the equation for the line of reflection? In patients infected with nonresistant strains of the tuberculosis bacterium, antibiotics can relieve symptoms in a few weeks. However, it takes much longer to halt the infection, and patients may discontinue treatment while bacteria are still present. How might this result in the evolution of drug-resistant pathogens? is the sole proprietor of Prestigious Pups, a business specializing in the sale of high-end pet gifts and accessories. Prestigious Pups' sales totaled $ 986 comma 000 during the most recent year. During the year, the company spent $ 58 comma 500 on expenses relating to website maintenance, $ 30 comma 500 on marketing, and $ 29 comma 100 on wrapping, boxing, and shipping the goods to customers. Prestigious Pups also spent $ 643 comma 000 on inventory purchases and an additional $ 19 comma 700 on freight-in charges. The company started the year with $ 16 comma 250 of inventory on hand and ended the year with $ 16 comma 800 of inventory. Prepare Prestigious Pups' income statement for the most recent year. If x+y+z=13 what is the value of (x-1)+(y+1)+(z-1) 293,671 in word from Juliet rented a car for one day from a company that charges $80 per day plus $0.15 per mile driven. If she was charged a total of $98 for the rental and mileage,for how many miles of driving was Juliet charged?(Assume there is no tax.)A) 15B) 120C) 533D) 633 Describe how to find the sums -4+2 and -4+(-2) on a number line. what is the probability of rolling an odd number on one roll of dice Employment data at a large company reveal that 74% of the workers are married, 42% are college graduates, and that 56% are married, given that they are college graduates. Which of the following statements are true about the events married and college graduate?(A) These events are pairwise disjoint.(B) These events are independent events.(C) These events are both independent and pairwise disjoint.(D) A worker is either married or a college graduate always.(E) None of these above are true. On January 1, 2016, Jacob Inc. purchased a commercial truck for $48,000 and uses the straight-line depreciation method. The truck has a useful life of eight years and an estimated residual value of $8,000. On December 31, 2018, Jacob Inc. sold the truck for $30,000. What amount of gain or loss should Jacob Inc. record on December 31, 2018? A.Loss, $3,000. B.Loss, $18,000. C.Gain, $22,000. D.Gain, $5,000. The car is traveling at a constant speed v0 = 80 km/hr on the level portion of the road. When the 4-percent (tan q = 4/100) incline is encountered, the driver does not change the throttle setting and consequently the car decelerates at the constant rate g sin q. Determine the speed of the car: (a) 28 seconds after passing point A ,and (b) when s = 620 m.